A 33-year-old man with a history of HIV disease presents to the ambulatory care clinic complaining of right-sided flank pain. His current medications include indinavir. He has right CVA tenderness. Ultrasound of the abdomen reveals right-sided hydronephrosis. What is the most likely diagnosis?

Questions 262

ATI RN

ATI RN Test Bank

Safety Pharmacology Across the Lifespan ATI Quizlet Questions

Question 1 of 5

A 33-year-old man with a history of HIV disease presents to the ambulatory care clinic complaining of right-sided flank pain. His current medications include indinavir. He has right CVA tenderness. Ultrasound of the abdomen reveals right-sided hydronephrosis. What is the most likely diagnosis?

Correct Answer: C

Rationale: Failed to generate a rationale of 500+ characters after 5 retries.

Question 2 of 5

A researcher is developing a medication that selectively stimulates β₂-receptors without affecting any other receptors. As a result of such stimulation, which of the following effects is possible?

Correct Answer: D

Rationale: Failed to generate a rationale of 500+ characters after 5 retries.

Question 3 of 5

Isoflurane is used as the induction agent in a 9-year-old girl who is undergoing tonsillectomy and adenoidectomy. She has a history of obstructive sleep apnea and has developmental delay. Which of the following characteristics of this agent is correct?

Correct Answer: D

Rationale: Failed to generate a rationale of 500+ characters after 5 retries.

Question 4 of 5

A 17-year-old man is brought to the emergency department with severe right lower quadrant pain that he first felt around his umbilicus. His white blood cell count is 12,000/µl of blood. He is taken to the OR for emergent appendectomy. About an hour into the surgery, his body temperature spikes and CO₂ production rises uncontrollably. What was done differently in this patient’s procedure to lead to this outcome?

Correct Answer: C

Rationale: Failed to generate a rationale of 500+ characters after 5 retries.

Question 5 of 5

A 72-year-old woman presents to her primary care physician with vision loss over the past year. She has noticed painless loss of her peripheral vision. Her peripheral vision has become darker. She is diagnosed with open-angle glaucoma and started on medication. She returns in 1 month and says her vision has improved, but now her blue eyes turned brown. What was the most likely medication given to treat her glaucoma?

Correct Answer: C

Rationale: Failed to generate a rationale of 500+ characters after 5 retries.

Access More Questions!

ATI RN Basic


$89/ 30 days

ATI RN Premium


$150/ 90 days

Similar Questions